Comprendre la logique des mathématiques

13468912

Réponses

  • Pour le 15 :
    Comment déduis-tu de ce que tu as écrit que $f$ n'est pas affine ?
    D'ailleurs, peux-tu donner une définition de fonction affine ?
  • Une fonction affine est une fonction $f: \R \longrightarrow \R \\ x \mapsto ax+b$ où $(a,b) \in \R^2$.

    La pente est constante vu que c'est une droite. Or j'ai trouvé 2 valeurs distinctes pour la pente. On voit que la courbe représentative de la fonction $x \mapsto x^2$ n'a pas une pente constante.
  • Pour s'y retrouver plus facilement, j'ai mis dans le premier message du fil un lien vers les exercices proposés (j'ai gardé la numérotation de christophe c).
    N'hésitez pas à me signaler des erreurs ou des oublis.
  • Ta définition manque de précision, $a$ et $b$ ne sont pas quantifiés.
    Tu dis : "La pente est constante vu que c'est une droite". Qui est une droite ?
  • La courbe représentative d'une fonction affine est une droite.

    $f$ est une fonction affine si : $\exists (a,b) \in \R^2 \ \forall x \in \R \ f(x)=ax+b$
  • Pour la 15 ok
    Pour la 2 ok mais tu peux simplifier en remarquant que $$|f'(x)|= \lim\limits_{y \rightarrow x} |\dfrac{f(y)-f(x)}{y-x}|\leq ...$$
    Le 😄 Farceur


  • Je suis héberlué par la vitesse de tes progrès!!! Bravo, et merci à tous les intervenants pourleur forte présence dans ce fil. Ca me laisse sans voix.

    Je vais t'ajouter dans les prochains temps plein d'exercices pour transformer l'essai, mais tu as déjà réussi des choses inattendues, je trouve.
    Aide les autres comme toi-même car ils sont toi, ils sont vraiment toi
  • OShine a écrit:
    Exercice 2.1 :

    Soit $x \in \R$. Soit $y > x$. Comme $f$ est dérivable sur $\R$, on a : $f'(x)= \lim\limits_{y \rightarrow x} \dfrac{f(y)-f(x)}{y-x}$

    Ta démonstration est à peu près bonne, mais mis à part une coquille à la fin (un 33 s'est transformé en un 3), il y a un défaut de rédaction. Tu commences par "soit $y>x$", c'est-à-dire que tu te donnes un nombre $y>x$. Mais si ensuite tu écris $f'(x)= \lim\limits_{y \rightarrow x} \dfrac{f(y)-f(x)}{y-x}$, ça ne va pas car le $y$ qui apparaît dans cette dernière expression est une variable muette. Pour que ta démonstration soit correcte, il suffit de supprimer le "soit $y>x$" du début qui ne sert à rien (puisque de toute façon tu dis plus loin "Pour $x<y$").
  • OK pour la 15. Et OK pour tes réponses à mes questions.
    J'en profite pour te féliciter, à mon tour, pour ces rapides progrès.

    Tu peux simplifier également pour la question 15. Sans utiliser le taux d'accroissement ni aucun résultat sur les fonctions affines. Par exemple :
    Si $f$ (la fonction carrée) est affine, alors il existe $a$ et $b$ tel que pour tout $x$ réel, $f(x)=x^2 = ax+b$.
    Pour $x=0$, on a d'une part $f(0)=a \times 0 + b = b$ et d'autre part $f(0)=0^2 = 0$. Donc $b=0$.
    pour $x=1$, on a d'une part $f(1)=a \times 1 + 0 = a$ et d'autre part $f(1)=1^2 = 1$. Donc $a=1$.
    Mais : $f(2)=1 \times 2 + 0 = 2$ et $f(2)=2^2 = 4$.
    Contradiction.
    $f$ n'est donc pas une fonction affine.
  • Michael une preuve rapide pour la 15. SI x-->x^2 était affine alors sa dérivée serait constante sur R.
    Le 😄 Farceur


  • Je donne aussi une correction pour la 15, encore plus formelle et froide, et j'ajoute des commentaires.

    Hypothèse1: pour tout $x: f(x)=ax+b$
    Hypothèse2: pour tout $x: f(x)=x^2$

    donc $f(0) = a\times 0 + b$
    donc $f(0) = 0+b$
    donc $f(0) = b$

    donc $0^2=b$
    donc $b=0$

    donc $f(1) = a\times 1+b$
    donc $f(1) = a+b$

    donc $1^2 = a+b$
    donc $1 = a+b$
    donc $1=a+0$
    donc $a=1$

    donc $f(2) = 2a+b$
    donc $f(2) = 2 \times 1+0$
    donc $f(2) = 2$

    donc $f(2) = 2^2$
    donc $f(2) = 4$

    donc $2=4$

    Commentaires:
    c'est encore imparfait, car j'ai mis "donc" partout (ou bien rien ou bien donc, et quand c'est rien, c'est une hypothèse), mais ces "donc" ne renvoient pas à la phrase d'avant, mais possiblement à des phrases qui se trouvent beaucoup plus haut dans la liste.

    Un point peut-être plus important: on a une situation

    $$ [\exists u:R(u)] \to P$$

    il est LOGIQUEMENT autorisé de considérer que ça veut dire la même chose (***) que:

    $$ \forall u: [(R(u))\to P]$$

    avec la restriction, bien entendu que $P$ ne doit pas parler de $u$.

    C'est une équivalence valable dans TOUTES LES LOGIQUES, même les plus faible.

    Aucun correcteur ne la refusera.

    L'inconvénient de rédiger en disant "comme [il existe a,b: Machin(a,b)],donc ... donc ... donc blabla(a,b)" est que $a,b$ ne sont jamais introduits du fait que $a,b$ dans les crochets sont liés (donc n'existent pas avec des valeurs).

    Classiquement, on peut corriger ce défaut par:

    "comme [il existe a,b: Machin(a,b)]. Soient $a,b$ ainsi. Donc ... donc ... donc blabla(a,b)"

    Mais c'est gaspillant en nombres de caractères.

    L'axiome logique parfaitement officiel (***) semble le plus naturel. Evidemment, sa légèreté vient de ce que $\exists$ est en polarité NEGATIVE. Ne pas confondre avec les $\exists $ de conclusion.

    Pour les gens qui uniquement des maths et n'utilisent que des rudiments de logique classique, ils peuvent le voir via le fait que:

    $$ [(\exists xR(x))\to A]\iff [(\forall x:(nonR(x)))\ ou \ A] $$

    et en fait prouveront le troisième truc: $\forall x: ( \ \ [nonR(x)]$ ou $A \ \ )$

    avec l'inconvénient que ça utilise deux choses puissantes et non intuitionnistes.
    Aide les autres comme toi-même car ils sont toi, ils sont vraiment toi
  • @gebrane : Ou sinon $ax+b = o(x^2)$ quand $x$ tend vers l'infini. Le problème avec ce que tu proposes et ce que je viens de donner c'est que c'est "plus cher en complexité" que ce qui est proposé par michael et CC (ils ont juste deux évaluations à faire). Toi tu utilises un calcul de limite en chaque réel $x$, et il faut ensuite vérifier que la fonction $x \mapsto 2x$ n'est pas constante. Certes c'est évident, mais le but est, je pense, de faire en sorte qu'OShine soit capable de faire ses propres raisonnements sans le moindre résultat théorique lorsque c'est possible, puisque ça ne fait qu'obscurcir la complexité du résultat.
  • Poirot
    Si on exige de Oshine que des raisonnements élémentaires basés que sur des évaluations sur des questions similaires, je crains pour lui si on lui pose .
    40 bis / On suppose que pour tout nombre $x:a_0+a_1x+....+a_{1000} x^{1000}=1+2x+3x^2+...+1000x^{1000}$. Peut-on en déduire que $a_0\times a_1\times ...\times a_{1000}=1\times 2 \times...\times 1000?$
    Le 😄 Farceur


  • La solution du 15) fournie par OShine convient puisque :$$f:\R\to\R\text{ est affine ssi il existe un réel }a\text{ tel que pour tous réels }x\neq y,\;\frac{f(x)-f(y)}{x-y}=a.$$
  • Cher gai requin, n'aurais-tu pas oublié de préciser « par théorème » ? :-D
  • Excellent brian ! ;-)
  • Merci pour vos solutions alternatives pour le 15.

    J'essaierai de résoudre un ou 2 exercices de plus dans la journée.
  • Le 2.2 il y a écrit "difficile" du coup ça me fait peur.
  • Pourtant rien ne doit faire peur : si tu peux appliquer une règle de maths, tu l’essayes.
    Ne pas trouver n’est pas grave.
    Ce qui doit te faire peur est d’appliquer un théorème sans en avoir le droit.
  • Oshine. Tu as démonté la 2.2 implicitement dans la 2.1. Reste à te convaincre pourquoi!
    Le 😄 Farceur


  • La limite du taux d'accroissement est finie, donc $f$ est dérivable et on retrouve le 2.2 à partir de la démonstration du 2.1.
  • Bravo.
    Cc à mis difficile pour te tester
    Le 😄 Farceur


  • Merci. Je repasse à l'exercice 4, j'avais des idées mais je n'ai trouvé encore. A voir si j'arrive à trouver une solution.
  • Attends, tu dis "la limite du taux d'accroissement est finie" mais pourquoi cette limite existe-t-elle ?
  • La limite existe d'après le théorème d'encadrement.
  • Je ne trouve pas le 4 j'ai tout essayé.
  • Tu peux utiliser l' algèbre linéaire ( théorème du rang) mais interdit par cc. Tu peux aussi t'inspirer de la géométrie. Ton problème se ramène à l'intersection de deux plans dans l'espace passant par l'origine. Je te rappelle que l’équation $$ax+by+cx=0$$ est celle dans plan dans l'espace passant par (0,0,0) si $(a,b,c)\neq (0,0,0)$ et tout l"espace si $(a,b,c)= (0,0,0)$ Je ne peux t'en dire plus !
    Le 😄 Farceur


  • De toute façon je ne vois pas comment faire avec l'algèbre linéaire même si ça ressemble à un noyau.

    Je tombe sur le système :

    $\begin{cases}
    ax+bz+cu = 0\\
    ay+bt+cv=0
    \end{cases}$
    Ce sont 2 plans vectoriels de vecteurs normaux $(x,z,u)$ et $(y,t,v)$. L'intersection est soit une droite soit vide si les plans sont parallèles

    Je vais continuer à chercher.
  • Il est horrible cet exercice.
    Je trouve comme solution, sous réserve que $x \ne 0$ et $zy-xt \ne 0$ :

    $a= -c (\dfrac{u}{x} + \dfrac{z}{x} \dfrac{vx-uy}{zy-xt})$ avec $c \in \R$ quelconque.
    $b=\dfrac{vx-uy}{zy-xt} c$ avec $c \in \R$ quelconque.
  • 1) Tu dis "L'intersection est soit une droite soit vide". Est-il possible que l'intersection soit vide ? Et est-il impossible que l'intersection soit autre chose qu'une droite ou l'ensemble vide ?

    2) Avec ta solution calculatoire : on te demande de prouver qu'il existe $(a,b,c)\ne (0,0,0)$ tel que (...) donc tu n'as pas fini l'exercice, il faut donner des valeurs de $a,b,c$ et vérifier qu'elles ne sont pas toutes nulles. D'autre part, ton expression de $a$ peut se simplifier et normalement on reconnaît quelque chose.
  • Une solution non constructive utilisant le théorème du rang est une approche niveau L1 qui me paraît très pertinente pour quelqu'un qui va passer les écrits du CAPES dans peu de temps.
    D'autant qu'ici une solution constructive est lourde à mettre en place.
  • Bonjour,

    Raisonner sans calculs en termes de droites, plans et produit scalaire me paraît le plus simple.

    Cordialement,

    Rescassol
  • Une solution constructive ne devrait pas poser de problème à un physicien.
  • Salut Rescassol,

    Pour paraphraser pappus, je dirais que le problème est vectoriel et qu'on peut (doit ?) donc lui apporter une solution vectorielle. ;-)

    @JLT : Je n'ai pas encore vu le produit très prisé des physiciens (il me semble, moi et la physique hein) qui règle l'affaire dans la plupart des cas...
  • En voici d'autres et MERCI pour le temps que vous mettez dans ces échanges.

    45/ Soit $A$ un ensemble de points du plan. On suppose que pour tout $X\subset A$ si $card(X)\leq 3$ alors il existe une droite $d$ telle que $X\subset d$. Prouver qu'il une droite $d$ telle que $A\subset d$.

    46/ Prouver avec uniquement les bases du collège et sans Thalès que les points $A,B,C$ étant alignés dans cet ordre, on a:

    $$ x_Ay_B = x_By_A $$

    le plan ayant été supposé muni d'un repère orthonormé [large]où le point $C$ a pour couple de coordonnées $(0,0)$[/large]


    47/ Soit $u$ une suite de nombres réels. Soit $v: n\mapsto u(2n)$ et $w: n\mapsto u(2n+1)$. Se peut-il que $u$ ne soit pas arithmétique et $v,w$ le soit?


    48/ Un élève de lycée a oublié la formule qui permet de calculer la dérivée du produit de deux fonctions (dérivables) mais se rappellent toutes les autres formules autorisées au bac sans justification. Peut-il théoriquement retrouver la formule oubliée?


    49/ Soit $A$ un ensemble de rationnels. Prouver qu'il existe $e\in A$ qui a la propriété suivante:

    pour tout $x\in \Q: [$ si $e\in A$ alors $x\in A]$


    50/ Soit $A$ l'ensemble des polynômes, enfin des fonctions polynomiales de $\R\to \R$. Soit $f\in A$ telle que pour toute $g\in A: f\circ g = g\circ f$. Peut-on déduire de ces hypothèses qui est $f(103)$?

    51/ Je ne suis pas dispo, mais j'ai lu que tu avais prouvé que $x^2+y^2\geq 2xy$ et ensuite déclaré que tu n'arrives pas à ne déduire
    $x^2+y^2 \geq xy$. Peut-être as-tu déjà surmonté la difficulté, mais sinon, peux-tu rédiger une solution totalement détaillée?
    Aide les autres comme toi-même car ils sont toi, ils sont vraiment toi
  • Je comente le (4) en traitant une dimension de moins. De plus, j'attire ton attention, OShine, sur le fait que je ne vais pas faire de divisions. Ce que tu fais beaucoup dans tes commentaires sur le 4 alors qu'elles ne sont pas nécessaires.


    Version light: on souhaite prouver l'existence de $(x,y)\neq (0,0)$ tel que $ax+by=0$. Sans rien supposer que $a,b$, ni même qu'on est dans un anneau intègre.

    Je SUPPOSE QUE $\forall (x,y)\neq (0,0): \ ax+by\neq 0$.
    Donc, du fait que $(-b)a + ab=0$, il découle $a=b=0$. Donc $1a+0b=0$, donc $1=0$.


    Penses-tu qu'un argument analogue et "tranquille" n'existerait pas pour le 4?
    Aide les autres comme toi-même car ils sont toi, ils sont vraiment toi
  • Christophe j'ai essayé cette méthode mais je n'ai pas réussi, la difficulté est qu'il y a 2 équations en parallèle.
    Je n'aime pas non plus diviser par des nombres alors que je ne sais pas s'ils sont nuls ou pas.

    Gai Requin je ne vois pas comment utiliser le théorème du rang.

    @JLT
    Pour 2 plans, soit ils sont confondus, soit ils sont parallèles soit ils se coupent.

    Je trouve une solution qui fonctionne en ayant continué mon raisonnement et en ayant judicieusement choisi $c$ qui était fixé dans $\R$ :

    $\boxed{(a \ b \ c )=(uxt-zvx \ \ , vx^2-uyx \ \ , zyx-x^2t)}$
  • Tu peux diviser ton triplet solution par $x$, et vérifier s'il marche toujours. Mais tu n'as toujours pas vérifié si $(a,b,c)\ne(0,0,0)$.
  • @OShine : Crée une application linéaire dont le noyau est défini par le système que tu cherches à résoudre.
  • Je vais peut être dire d'énormes bêtises :

    $f : \R^3 \longrightarrow \R^2 \\ (a ,b,c) \mapsto (ax+bz+cu, ay+bt+cv)$

    $\dim \ker(f)+ \dim Im(f)= 3$
    Je ne connais pas la dimension de l'image.
  • Tu ne connais pas la dimension de l'image, mais tu sais qu'elle est plus petite que...
  • Si $a=ut-zv$ , $b=vx-uy$ et $c=zy-xt$ je ne vois pas comment montrer que les composantes sont non toutes nulles alors que je ne connais rien sur les variables.
  • @OShine : Que dois-tu montrer pour avoir $(a,b,c)\neq (0,0,0)$ répondant au problème de cc ?
  • Je ne comprends pas à quoi sert l'application linaire et l'utilisation du théorème du rang pour résoudre le problème.

    On doit montrer $ut-zv \ne 0$ et $vx-uy \ne 0$ et $zy-xt \ne 0$
  • Pourquoi ton application $f$ ne peut pas être injective ?
  • cc pour ta solution dans http://www.les-mathematiques.net/phorum/read.php?16,2003520,2022988#msg-2022988
    Tu as triché et maquillé ton raisonnement pour dissimiler la vraie problématique.
    Montrons montrons qu'il existe $(x,y)\neq (0,0)$ tel que $ax+by=0$
    Raisonnement à la gebrane:
    Si $(-b,a)\neq (0,0)$ rien à prouver
    Si $(-b,a)= (0,0)$, alors (1,1) convient ( en fait n'importe quel coupe non nul convient)
    Ce que tu as dissimuler cc c'est que tu as déjà un couple solution. Pour ce qui nous concerne, peux-tu nous donner sans peiner un triplet (x,y,z) " évitons quand même (0,0,0)" solution de $ax+by+xz=0,\quad a'x+b'y+x'z=0$
    Le 😄 Farceur


  • $f$ est injective si et seulement si $Ker (f) = \{0 \}$

    Or $\dim \{0 \}=0$ ce qui signifierait que $\dim Im (f)=3$ ce qui est impossible car $f$ est à valeurs dans $\R^2$.

    On doit avoir $\dim Im(f) \leq 2$ car l'image est inclue dans l'ensemble d'arrivée.
  • Conclusion ?
  • OShine a écrit:
    Si $a=ut-zv$ , $b=vx-uy$ et $c=zy-xt$ je ne vois pas comment montrer que les composantes sont non toutes nulles
    OShine a écrit:
    On doit montrer $ut-zv \ne 0$ et $vx-uy \ne 0$ et $zy-xt \ne 0$

    Attention, "non toutes nulles" $\neq$ "toutes non nulles".
  • Comme :

    $Ker(f)= \{ (a,b,c) \in \R^3 | ax+bz+cu=0 \ \text{et} \ ay+bt+cv=0 \}$

    Et $Ker(f) \ne 0$ alors le triplet $(a,b,c)$ solution n'est pas le triplet nul.

    Merci je viens de comprendre la logique ;-)
Connectez-vous ou Inscrivez-vous pour répondre.